Use the graph below to describe the linearization of the data. How would you expect the linearization to change if the data were to extend beyond age 20?

PLEASEEEE HELPPPP!!

Use The Graph Below To Describe The Linearization Of The Data. How Would You Expect The Linearization

Answers

Answer 1

Answer:

The coordinates of the points in the graph are;

(5, 2), (10, 3.5), (15, 5), 20, 5.5)

The data points are plotted using MS Excel

Given that the height of a person stops increasing beyond around 20 years of age, it is expected that the height will remain slightly the same at 5.5 feet beyond 20 as follows;

(25, 5.5), (30, 5.5), (35, 5.5), (40, 5.5)

Therefore, if the data were to extend beyond 20, the linearization will give a more horizontal line with a gentler slope

Please see attached graphs created with MS Excel

Step-by-step explanation:

Use The Graph Below To Describe The Linearization Of The Data. How Would You Expect The Linearization
Use The Graph Below To Describe The Linearization Of The Data. How Would You Expect The Linearization

Related Questions

Find the IQR and the Spread to this graph

Answers

Answer:

1 - 4

Step-by-step explanation:

26/4 = 6.5

IQR is the 7 - 19 item

1 - 4

How do you write 9% as a decimal?

Answers

9 over 100
=0.09
That should be the answer

Answer:

0.09

Step-by-step explanation:

9% written as a decimal is 0.09

8%=0.08

7%=0.07

6%=0.06

5%=0.05

4%=0.04

3%=0.03

2%=0.02

1%=0.01

FOR EASY BRAINLIEST:

ANSWER NUMBER: 14.

Answers

Answer:

Step-by-step explanation:

Answer:

y=-3/2x+4

Step-by-step explanation:

The hypotenuse of a right angle triangle is 50cm and the legs are bin the ratio of 7:24. Find the area of right angle triangle.

Answers

Answer:

84 cm^2

Step-by-step explanation:

area of right triangle formula= 1/2 (base*height)

In order to find the area, you need the base and the height

The question included that the legs ratio are 7 to 24.

If you do [tex]\sqrt{7^2+24^2}[/tex], you get 25 (Pythagorean theorem)

so the base and heights are 7 and 24

1/2 (7*24)= 84

Determine the equation of a vertical line that passes through the point (-5,-2).

Answers

Answer:

Step-by-step explanation:

A vertical line is an "x = " equation. To find out what this equation is, just plug in the value that satisfies the x value in the coordinate:

x = -5

If we were looking for the horizontal line that goes through that same point, we only need remember that a horizontal line is a "y = " line. To find out what this equation is, just plug in the value that satisfies the y value in the coordinate:

y = -2

A high school football team has raised $1000 to spend on team jackets. The cost is $50 per jacket. Which equation below can be used to solve for the number of jackets the team can buy? Explain.
A. 50 = 1000n
B. 50n = 1000
C. 1000n = 50n

Answers

B. 50n =1000
n is for the number of jackets so n times 50 per jacket equals the $1000 the team raised

david study table is 20 inches long. if the diagonal measures 25 inches, find the width of david study table

Answers

Answer:

15 inches

Step-by-step explanation:

Since we know diagonal and a side of a right triangle, we can use the Pythagorean theorem to solve.

a^2 + b^2 = c^2  where a and b are the legs and c is the hypotenuse

20 ^2 + b^2 = 25^2

400+b^2 =625

b^2 =625 -400

b^2 =225

Taking the square root of each side

sqrt(b^2) = sqrt(225)

b= 15

Answer:

15

Step-by-step explanation:

You can use Pythagorean theorem or if you draw it out and label the given information you may recognize that the triangle created is a 3 - 4 - 5 triangle.

[tex]25^{2}[/tex] = [tex]20^{2}[/tex] + [tex]x^{2}[/tex]             [tex]c^{2}[/tex] = [tex]a^{2}[/tex] + [tex]b^{2}[/tex]

625 = 400 + [tex]x^{2}[/tex]        

225 = [tex]x^{2}[/tex]                     Square root both sides

15 = x

A special right triangle has a leg that measures 3, another leg that measures 4. and a hypotenuse that measures 5.

The given triangle in this problem has a leg that measures 20 and a hypotenuse that measures 25. If you divide each measure by 5, you will have a leg that is 4, and a hypotenuse that is 5. That means the last leg must be 15.

Why? Because 3 x 5 = 15. or 15/5 = 3

This comes up very often when working with triangles.

Use the shell method to write and evaluate the definite integral that represents the volume of the solid generated by revolving the plane region about the y-axis.


y = x^3/2,

y=8

x=0

Answers

Solution :

[tex]$y=x^{3/2}, \ \ y =8, \ \ x=0$[/tex]

[tex]$v=2 \pi \int_0^4 x (8-x^{3/2}) \ dx$[/tex]

 [tex]$= 2\pi \left(\frac{8x^2}{2}-\frac{x^{7/2}}{7/2}\ \left|_0^4[/tex]

 [tex]$=2\pi \left( 4x^2-\frac{2}{7}x^{7/2}\ |_0^4$[/tex]

 [tex]$=2\pi \left( 64-\frac{2}{7} \times 32 \times 4 \right)$[/tex]

  [tex]$=2\pi \left(1-\frac{4}{7} \right) 64$[/tex]

  [tex]$=128 \pi \times \frac{3}{7}$[/tex]

 [tex]$=\frac{384 \pi}{7}$[/tex]

Transversal Problems with Equations (Level 1)
Jol 19, 2:05:45 PM
Given m||n, find the value of x.
(8x-7)°
(x-28)
Answer:
Submit Answer

I need help

Answers

Answer:

x is 21°

Step-by-step explanation:

The angles formed between the transversal t and lines m and n are; (8·x - 7)°, and (9·x - 28)°

Based on the similar location the angles are formed by the transversal, t, and lines m and n, the angles are corresponding angles

Given that lines, m and n are parallel, we have the corresponding angles formed by the transversal, t, and the lines are equal, therefore;

(8·x - 7)° = (9·x - 28)°

Simplifying the above equation to make x the subject, we get

(28 - 7)° = 9·x - 8·x = x

∴ 21° = x

x = 21°.

11 red marbles, 9 blue marbles and 20 green marbles as a ratio in its simplest form

Answers

Answer:

11 red

9 blue

20 green

as a ration is

11:9:20

since 9 and 11 are even numbers and can't be simplified

400th answer

Hope This Helps!!!

(2.35 x 10^4) – (4.50 x 10^5)

Answers

Answer:

−426500

Step-by-step explanation:

the scale for a certain model freight train is 1: 48. if the model hopper car (usually used for carrying coal) will hold 90 in3 of coal, what is the capacity of the actual hopper car?​

Answers

Answer:

Step-by-step explanation:

Scale measure is a one-to-one thing while volume is a cubed thing. If we want to find the volume that the actual car can hold, we have to find the volume that results from cubing the one-to-one measure.

[tex]\frac{m}{a}:\frac{1^3}{48^3}=\frac{1}{110592}[/tex] That last fraction is the ratio of the volumes. Remember, volume is a cubed measure (where distance is a linear measure and area is a squared measure. We find ratios of areas by squaring the one-to-one {distance} measures, and we find ratios of volumes by cubing the one-to-one measures). We have a given volume for the model (m in our ratio) and we are looking for the capacity that the actual car (a in our ratio) can hold. Set up the proportion as follows:

[tex]\frac{1}{110592}=\frac{90}{x}[/tex] and cross multiply

x = 9,953,280 inches cubed

HELP!!! 15 points. picture below

Answers

Step-by-step explanation:

[tex] \sin( \alpha ) = \frac{ \sqrt{5} }{3} \\ \alpha = 48.19 \: degrees \\ \cos( \alpha ) = \frac{2}{3} [/tex]

Answer:

cos theta =± 2/3

Step-by-step explanation:

sin theta = sqrt(5) /3

sin theta = opp side / hypotenuse

We know that

a^2 + b^2 = c^2 from the pythagorean theorem

opposite side ^2 + adjacent side ^2 = hypotenuse ^2

( sqrt(5)) ^2 +   adjacent side ^2  = 3^2

5 +  adjacent side ^2  = 9

Subtract 5 from each side

5-5 +  adjacent side ^2  = 9-5

 adjacent side ^2  = 4

Taking the square root of each side

 adjacent side   = ±2

We know that

cos theta = adj side / hyp

cos theta =± 2/3

True or false..?

In a parallelogram, consecutive angles are supplementary.

Answers

Answer:

True

Step-by-step explanation:

Both pairs of opposite angles are congruent. parallelogram, rectangle, rhombus, square. Both pairs of opposite sides are congruent. parallelogram, rectangle, rhombus, square. All consecutive angles are supplementary. parallelogram, rectangle, rhombus, square. diagonals bisect each other. parallelogram, rectangle, rhombus, square.

Answer:

true

Step-by-step explanation:

any 2 consecutive angles are supplamentary

David is going to an amusement park. The price of admission into the park is $20,
and once he is inside the park, he will have to pay $5 for every ride he rides on. How
much
money would David have to pay in total if he goes on 10 rides? How much
would he have to pay if he goes on r rides?

Answers

Answer:

$70.00

y=5r+20

Step-by-step explanation:

1. y=5(10)+20

y=70

The cost he will pay for 10 rides will be $70 and for r rides will be [tex]20+5r[/tex].

Given,

The price of admission into the park is $20.

The price for each ride is $5

David takes 10 rides,

So, the cost he will pay if he took 10 rides will be,

[tex]c_1=20+5 \times 10\\c_1=20+50\\c_1=70$[/tex]

If he had r rides,

So, the cost he will pay if he took r rides will be,

[tex]c_2=20+5 \times r\\c_2=20+5r[/tex]

Therefore, the cost he will pay for 10 rides will be $70 and for r rides will be[tex]20+5r[/tex].

For more details about linear equation, refer to the link:

https://brainly.com/question/3997793

Help Please
Find The Surface Area Of The Prism

Answers

Answer:

360 cm^2

Step-by-step explanation:

Find the area of each face of the triangular prism.

Imagine the triangular prism as its net, it is composed with 2 triangular faces (these are tye bases of the prism) and 3 rectangular faces.

Areas of the 2 triangular bases (they are similar triangles):

1/2 x 8 cm x 6 cm = 24 cm^2

24 x 2 = 48 cm^2

Area of the rectangular face:

8 x 13 = 104 cm^2

Area of another rectangular face:

6 cm x 13 cm = 78 cm^2

Area of another rectangular face:

13 cm x 10 cm = 130 cm^2

Add up all the areas of all faces:

48 + 104 + 78 + 130 = 360

So the SA is 360 cm^2

Polynomial x²-6y²-xy-5x-5y+6 was factored into (x+ay+b)(x+cy-2) for constants a, b and c. Find the value of a+bc

Answers

Answer:

We know that the polynomial:

x²- 6y²- xy - 5x - 5y + 6

is rewritten as:

(x+ay+b)(x+cy-2)

First, lets expand the above expression:

x^2 + x*(cy) + x*(-2) + (ay)*x + (ay)*(cy) + (ay)*(-2) + b*x + b*(cy) - 2*b

Now we can simplify this to get:

x² + c*(xy) - 2*x + a*(xy) + ac*y² - 2a*y + b*x + bc*y  - 2b

now let's group together the terms with the same variables:

x² + (c + a)*(xy) + (b - 2)*x + (bc - 2a)*y + ac*y² - 2b

And that must be equal to:

x²- 6y²- xy - 5x - 5y + 6

notice that equations are equal if and only if all the correspondent factors are equal.

notice that in both cases, the factor that multiplies the x² term is 1.

for the y² term we will have:

a*c = -6

for the xy term we will have

c + a = -1

for the x term we will have

b - 2  = - 5

for the y term we will have

bc - 2a = -5

for the constant term, we will have:

-2b = 6

Then we have a lot of equations, rewriting these we have:

a*c = -6

c + a = -1

b - 2  = -5

bc - 2a = -5

-2b = 6

From the fourth equation, b - 2  = -5

we can get:

b = -5 + 2 = -3

b = -3

notice that for the last equation:

-2b = 6

b = 6/-2 = -3

we have the same solution

Then we can replace the value of b in the above equations to get:

a*c = -6

c + a = -1

-3*c - 2a = -5

Now, we need to isolate one of the variables in one of the equations.

For example, we can isolate c in the second one to get:

c = -1 - a

now we can replace that in other equation, for example the third one:

-3*(-1 - a) - 2a = -5

now we can solve that for a.

3 + 3a - 2a = -5

3 + (3 - 2)a = -5

3 + a = -5

a =  -5 - 3 = -8

a = -8

now we can use the equation "c = -1 - a" to find the value of c:

c = -1 -(-8) = -1 + 8 = 7

c = 7

then we have:

b = -3

a = -8

c = 7

then:

a + b*c = -8 + (-3)*7 = -8 - 21 = -29

I have 15.00 and I want to go to the movies tickets cost 12.35 how much will I get back plz help

Answers

Answer:

2.65

Step-by-step explanation:

15.00 - 12.35 = 2.65

...I don't really know how do explain it but yeah thats the answer.

Hope it helps c:

Find the volume of the composite figure. Round to the nearest tenth. (Hint: Volume of a cone is V= 1/3 pie^2 h)

Answers

Answer:

the answer is in the pic so like yea hope that helps

For a given event, what is the result of dividing the number of successful
outcomes by the number of possible outcomes?
A. Outcomes
ОО
B. Probability
h
c. Sample space
D. Empirical data

Answers

Answer:  B. Probability

For example, let's say you want to know the probability of flipping tails.

There's 1 way to get tails out of 2 sides total. So 1/2 = 0.5 is the probability of flipping tails.

We define "success" as "getting tails".

If f(x) =3x^2 +1 and g(x) = 1 -x, what is the value of (f-g) (2)?

Answers

Answer:

(f-g)(2) = 14

Step-by-step explanation:

f(x) =3x^2 +1 and g(x) = 1 -x

f(2) = 3(2)^2 +1 = 3(4)+1 = 12+1 = 13

g(2) = 1-2 = -1

f(2) - g(2) = 13 - -1 = 13+1 =14

Answer:

14

Step-by-step explanation:

(f-g)(2) means f of x minus g of x when x equals 2.

To solve, first set up the equation

[tex](3x^2}+1)-(1-x)[/tex]

Change the signs in the second part. {because this is subtraction}

[tex]3x^2}+1-1+x[/tex]

Replace x with 2.

[tex]3(2^2})+1-1+2[/tex]

Solve.

[tex]3(4)+2[/tex]

[tex]12+2[/tex]

[tex]14[/tex]

Tìm min: 4 căn x -5/ 3 căn x+2

Answers

Answer:

bgyuhjvy;uhclgcxfghvmdxultgjvffgxfbc hgv

Step-by-step explanation:

vgfdhvczczxcgfzdsdvcxfdghcbcd

Shannon buys a table that was priced $700. There is a 8% sales tax in her state. Fortunately, it was 60% off! So she only paid

Answers

Answer:

Total paid: 302.40

Step-by-step explanation:

Price x percent off = amount off

700 x .60 (60%) = 420 amount off

price - amount off = sales price

700 - 420 = 280 sales price

sales price x sales tax rate = sales tax

280 x .08 (8%) = 22.40 sales tax

sales price + sales tax = total paid

280 + 22.40 = 302.40 total paid

Find the missing part.
Z=

Answers

Answer:

Step-by-step explanation:

To answer this, we need only solve for x and then use Pythagorean's Theorem to find z. We have the length of the hypotenuse which is 15, and we also have the base angle of 60. To solve for x we can use the trig ratio

[tex]cos60=\frac{x}{15}[/tex] and

x = 15cos60 which gives us that

x = [tex]\frac{15}{2}[/tex]

Now we can use Pythagorean's Theorem to find z, the other leg of the big triangle.

[tex](\frac{15}{2})^2+z^2=15^2[/tex] and

[tex](\frac{225}{4})+z^2=225[/tex] and

[tex]z^2=225-\frac{225}{4}[/tex] and getting a common denominator:

[tex]z^2=\frac{900}{4}-\frac{225}{4}[/tex] and

[tex]z^2=\frac{675}{4}[/tex] now we take the square root by simplifying the radical to get

[tex]z=\frac{15\sqrt{3} }{2}[/tex]

and that's what goes into the blanks there, one number per blank.

Which of the two-dimensional cross sections listed below could be created by cutting a cube with a plane?

Select all that apply.


pentagon

circle

square

hexagon

rectangle

triangle

Answers

rectangle and square

The function f(x) = log4x is dilated to become g(x) = f (1/3x).
What is the effect on f(x)?

Answers

Given:

The functions are:

[tex]f(x)=\log_4x[/tex]

[tex]g(x)=f\left(\dfrac{1}{3}x\right)[/tex]

The function f(x) is dilated to become g(x).

To find:

The effect on f(x).

Solution:

Transformation is defined as:

[tex]g(x)=f(kx)[/tex]            ...(i)

Where, k is the factor of horizontal stretch and compression.

If 0<k<1, then the graph of f(x) stretched horizontally by factor [tex]\dfrac{1}{k}[/tex].

If k>1, then the graph of f(x) compressed horizontally by factor [tex]\dfrac{1}{k}[/tex].

It is given that

[tex]g(x)=f\left(\dfrac{1}{3}x\right)[/tex]          ...(ii)

On comparing (i) and (ii), we get

[tex]k=\dfrac{1}{3}[/tex]

Therefore, the graph of f(x) stretched horizontally by factor [tex]3[/tex].

Identify the slope and y-intercept of each linear function's equation.
y = 3x - 1
slope = 3; y-intercept at -1
x-3=y
slope = -3; y-intercept at 1
y = 1 - 3x
slope = 1; y-interceptat -3
--x+ 3 = y
t
slope = -1; y-intercept at 3

Answers

Step-by-step explanation:

Concerning the peculiar interrogate, I will be providing correction(s) to the following answers inserted:

Y = 3x - 1

Slope = 3

Y-intercept = -1

X - 3 = y

Y = x - 3 <== Slope-Intercept Form.

Slope = 1

Y-intercept: -3

Y = 1 - 3x

Slope: -3

Y-intercept: 1

-x + 3 = y

Y = -x + 3 <== Slope-intercept Form.

Slope: -1

Y-intercept: 3

Thus, the following configurations have been defined or derived from the origin of the proposed interrogated.

*I hope this helps.

Will mark most brainly

Answers

Answer:

19 and 157

Step-by-step explanation:

6/9 as a decimal rounded to 3 decimal places

Answers

Answer: 0.667

Step-by-step explanation:

6/9 ⇔ Given

2/3 ⇔ Simplify fraction

0.666... (repeating) ⇔ Convert fraction to decimal

0.667 ⇔ Round to 3 decimal (Since 6 is more than 5, thus we need to

               rounding up)

Hope this helps!! :)

Please let me know if you have any questions

write and expression for the perimeter of a rectangle with length L and width 6

Answers

Answer:

P = 2(L + 6)

Step-by-step explanation:

The perimeter (P) of a rectangle is = 2(Length + Width)

Length = L

Width = 6

.: P = 2(L + 6)

2L + 2(6)=P
The L gets multiplied by 2 because there are two L in a rectangle and 2 times 6 because there are two widths
Other Questions
Marietta is visiting her company's manufacturing subcontractor in Pakistan. She notices children working on the machinery with no air conditioning and little access to water. But she also knows these children are probably supporting their families. Marietta can't decide if children working in the factory is appropriate or inappropriate. This is an example of a(n) ______. how much ounces are in 22 grams 0.035x Can someone help me please Ram thinks of a number if he adds 3 to 5 times the number he gets 38. what is the number? Write a program that asks the user to enter 5 test scores. The program will display a letter grade for each test score and an average grade for the test scores entered. Three functions are needed for this program. PLS HELP WILL MARK BRAINLIEST On a piece of paper, draw a box plot to represent the data. Then determinewhich answer choice matches the box plot you drew.12, 14, 22, 23, 25, 28, 32, 37, 38, 39, 41 What are the coordinates of the midpoint between the points (-4, -9) and (-8, -5)? Select the best answer from the choices provided. A. (-4, -4) B. (-6, -7) C. (-6, -4) D. (-4, -7) Verify that the indicated family of functions is a solution of the given differential equation. Assume an appropriate interval I of definition for each solution. d^2y/ dx^2 6 dy/dx + 9y = 0; y = c1e3x + c2xe3x When y = c1e3x + c2xe3x, 11.(02.03)The table shows the solution to the equation |2x - 31 - 1 = 2:Step 1 |2x - 3) = 2 + 1Step 2 12x - 3| = 3Step 3 2x - 3 = 3 or 2x + 3 = 3Step 4 2x = 6 or 2x = 0Step 5 x = 3 or x = 0Which is the first incorrect step? (1 point)O Step 1O Step 3Step 5O Solution is correct TJs Cat Food plans to use tins that are the shape of cylinders. The internal measurements of a tin are shown.(b ) Work out the volume of cat food that the tin contains. Please help!!! Find the domain of the function y = 2 cot(58x).A) All real numbers except odd integer multiples of 85B) All real numbers except 0 and integer multiples of 85C) All real numbers except 0 and integer multiples of 45D) All real numbers except odd integer multiples of 45 what is meaning of being savage These states Do Not have stand your ground laws A) Connecticut b) Oregon c) New York d ) Arkansas e) California Simplify to the extent possible(logx16)(log2x) Sammy Co. uses process costing to account for the production of popcorn. Direct materials are added at the beginning of the process and conversion costs are incurred uniformly throughout the process. Equivalent units have been calculated to be 20,000 units for materials and 18,200 units for conversion costs. Beginning inventory consisted of $6,200 in materials and $4,400 in conversion costs. May costs were $47,000 for materials and $32,000 for conversion costs. Ending inventory still in process was 4,000 units (100% complete for materials, 55% for conversion). The cost per equivalent unit for conversion costs using the weighted average method would be: Find f(-2) for f(x) = 23^x . 1/18O B. -18O C. -36OD. 2/9 Can you find a strategy for splitting any number so that you always get the largest product? Would you favor spending more federal tax money on the arts? Of a random sample of n1 = 222 women, r1 = 51 responded yes. Another random sample of n2 = 174 men showed that r2 = 49 responded yes. Does this information indicate a difference (either way) between the population proportion of women and the population proportion of men who favor spending more federal tax dollars on the arts? Use ???? = 0.05. How many tens are in 6 hundreds 5. What happens when William arrives at Mr. Godsten's welding shop with his bicycle and his shock absorber? How does William react to this experience? (the boy harnessd the wind)